Diễn Đàn MathScope

Diễn Đàn MathScope (http://forum.mathscope.org/index.php)
-   2012 (http://forum.mathscope.org/forumdisplay.php?f=175)
-   -   Việt Nam Team Selection Test 2012 - Đề bài, lời giải và danh sách đội tuyển (http://forum.mathscope.org/showthread.php?t=30511)

n.v.thanh 15-04-2012 07:35 PM

Việt Nam Team Selection Test 2012 - Đề bài, lời giải và danh sách đội tuyển
 
[Only registered and activated users can see links. Click Here To Register...]

Đến hẹn lại lên,sau kì thi VMO 2011 diễn ra cách đây hơn 2 tháng thì vào ngày mai và ngày kia Bộ GD và ĐT sẽ tổ chức kì thi chọn đội tuyển VN tham dự kì thi Olympic Toán quốc tế năm 2012 diễn ra ở Argentina .Sáng nay , 42 thí sinh đến từ khắp miền Nam Bắc đã tụ họp ở ĐHSP Hà Nội nghe điểm danh và phổ biến những điều cần biết. Sáng mai sẽ tiến hành thi luôn, thời gian vẫn là 4h30' cho mỗi ngày, với 3 bài toán.

Mình mở topic này để mọi người post đề bài, thảo luận về kì thi này. Vậy, mai ai có đề thì gõ Latex sạch đẹp và viết lên #2 nhé, chú ý là #2 chỉ để post đề bài, vi phạm thì mình buộc phải xoá.

Đây là một số hình ảnh mà mình lượm nhặt được trong buổi hôm nay
:





NhamNgaHanh 16-04-2012 12:54 PM

1 Attachment(s)
Ngày 1 - 16/4/2012
Thời gian làm bài 4h30'



Bài 1 (7 điểm)

Cho đường tròn $(O)$ và 2 điểm cố định $B,C$ trên đường tròn sao cho $BC$ không là đường kính của $(O)$, $A$ là một điểm di động trên đường tròn, $A$ không trùng với $B,C$. Gọi $D,K,J$ lần lượt là trung điểm của $BC,CA,AB$ và $E,M,N$ lần lượt là hình chiếu vuông góc của $A,B,C$ trên $BC, DJ, DK$.Chứng minh rằng các tiếp tuyến tại $M,N$ của đường tròn ngoại tiếp tam giác $EMN$ luôn cắt nhau tại $T$ cố định khi A thay đổi.

Bài 2 (7 điểm)

Trên một cánh đồng hình chữ nhật kích thước $m\times n$ ô vuông gồm $m$ hàng và $n$ cột người ta đặt một số máy bơm nước vào các ô vuông. Biết rằng mỗi máy bơm nước có thể tưới nước cho các ô vuông có chung cạnh với nó và các ô vuông cùng cột với nó và cách nó đúng một ô vuông . Tìm số nhỏ nhất các máy bơm nước sao cho các máy bơm nước có thể tưới hết cả cánh đồng trong 2 trường hợp:
a) $m=4$
b) $m=3$

Bài 3 (7 điểm)

Cho số nguyên tố $p\ge 17$. Chứng minh rằng $t=3$ là số nguyên dương lớn nhất thỏa mãn điều kiện: Với các số nguyên bất kì $a,b,c,d$ sao cho $abc$ không chia hết cho $p$ và $a+b+c$ chia hết cho $p$ thì tồn tại các số nguyên $x,y,z$ thuộc tập $\{0;1;...;\left[\frac{p}{t}\right]-1\}$ sao cho $ax+by+cz+d \, \vdots \,p$

Nguồn VMF

Bản Latex by CSS_MU.

thanhtra_dhsp 16-04-2012 02:54 PM

Sáng nay thi rồi mà, sao vẫn chưa có đề nhỉ?

thefallen 16-04-2012 06:01 PM

[Only registered and activated users can see links. Click Here To Register...]
Mod up lên dùm em!

n.t.tuan 16-04-2012 06:54 PM

Câu 2 trong file pdf lỗi nhiều quá Thanh kìa.

kien10a1 16-04-2012 07:46 PM

Em giải vắn tắt bài 1:
Gọi H là trực tâm tam giác ABC, ta thấy D,E,M,N,H đều thuộc đường tròn đường kính DH.
-,Đường thẳng qua H song song với BC cắt DO tại S. Ta chứng minh DMSN là tứ giác điều hòa.
Ta có $\frac{DM}{DN}=\frac{a\cos C}{a\cos B}=\frac{\cos C}{\cos B} $
$\frac{SM}{SN}=\frac{\sin SNM}{\sin SMN}=\frac{\sin SDM}{\sin SHN}=\frac{\cos C}{\cos B} $
Vậy T thuộc DO.
-, Hạ HX vuông góc AD( X thuộc AD), ta chứng minh XMEN là tứ giác điều hòa.
Nhờ sự đồng viên của E,D,K,J ta sẽ có $\Delta ENK\sim \Delta EMJ(g.g)\Rightarrow \frac{EM}{EN}=\frac{EJ}{EK}=\frac{AB}{AC} $
Cũng có:$\frac{XM}{XN}=\frac{\sin XNM}{\sin XMN}=\frac{\sin XDM}{\sin XDN}=\frac{\sin XAC}{\sin XAB}=\frac{AB}{AC} $
Vậy T thuộc EX.
T chính là giao của OD và EX

XH cắt BC tại Y thì theo một tính chất quen thuộc $(YEBC)=-1 $
Ta có $DT=\frac{DE}{\tan DET}=\frac{DE}{\tan XTD}=\frac{DE}{TX}.DX=\frac{AD.DX}{AH}=\frac{DE.DY }{2OD}=\frac{DB^2}{2OD} $, suy ra DT không đổi, suy ra ĐPCM

n.v.thanh 16-04-2012 07:51 PM

Bài hình nhìn giống IMO 2009 nhỉ :gach:.

novae 16-04-2012 08:37 PM

Trích:

Nguyên văn bởi kien10a1 (Post 144921)
Em giải vắn tắt bài 1:
Gọi H là trực tâm tam giác ABC, ta thấy D,E,M,N,H đều thuộc đường tròn đường kính DH.
-,Đường thẳng qua J song song với BC cắt DO tại S. Ta chứng minh DMSN là tứ giác điều hòa.
Ta có $\frac{DM}{DN}=\frac{a\cos C}{a\cos B}=\frac{\cos C}{\cos B} $
$\frac{SM}{SN}=\frac{\sin SNM}{\sin SMN}=\frac{\sin SDM}{\sin SHN}=\frac{\cos C}{\cos B} $
Vậy T thuộc DO.
-, Hạ HX vuông góc AD( X thuộc AD), ta chứng minh XMEN là tứ giác điều hòa.
Nhờ sự đồng viên của E,D,K,J ta sẽ có $\Delta ENK\sim \Delta EMJ(g.g)\Rightarrow \frac{EM}{EN}=\frac{EJ}{EK}=\frac{AB}{AC} $
Cũng có:$\frac{XM}{XN}=\frac{\sin XNM}{\sin XMN}=\frac{\sin XDM}{\sin XDN}=\frac{\sin XAC}{\sin XAB}=\frac{AB}{AC} $
Vậy T thuộc EX.
T chính là giao của OD và EX

XH cắt BC tại Y thì theo một tính chất quen thuộc $(YEBC)=-1 $
Ta có $DT=\frac{DE}{\tan DET}=\frac{DE}{\tan XTD}=\frac{DE}{TX}.DX=\frac{AD.DX}{AH}=\frac{DE.DY }{2OD}=\frac{DB^2}{2OD} $, suy ra DT không đổi, suy ra ĐPCM

Ta có thể chứng minh các tứ giác $DMSN$ và $XMEN$ điều hòa mà không cần thực hiện một phép tính nào:
Vì $HS \parallel BC$ và $D$ là trung điểm $BC$ nên $H(SDBC)=-1$ hay tứ giác $DMSN$ điều hòa.
Gọi $M',N'$ là chân các đường cao tương ứng với đỉnh $B,C$ của $\triangle ABC$. Khi đó dễ có tứ giác $AM'XN'$ điều hòa. Suy ra $H(XEBC)=H(XAM'N')=-1$ hay tứ giác $XMEN$ điều hòa.

kien10a1 16-04-2012 08:43 PM

Trích:

Nguyên văn bởi novae (Post 144928)
Ta có thể chứng minh các tứ giác $DMSN$ và $XMEN$ điều hòa mà không cần thực hiện một phép tính nào:
Vì $HS \parallel BC$ và $D$ là trung điểm $BC$ nên $H(SDBC)=-1$ hay tứ giác $DMSN$ điều hòa.
Gọi $M',N'$ là chân các đường cao tương ứng với đỉnh $B,C$ của $\triangle ABC$. Khi đó dễ có tứ giác $AM'XN'$ điều hòa. Suy ra $H(XEBC)=H(XAM'N')=-1$ hay tứ giác $XMEN$ điều hòa.

Em cắm cúi làm, thấy nó điều hòa là dùng ngay tỉ số :sad:

novae 16-04-2012 08:47 PM

Trích:

Nguyên văn bởi kien10a1 (Post 144921)
XH cắt BC tại Y thì theo một tính chất quen thuộc $(YEBC)=-1 $
Ta có $DT=\frac{DE}{\tan DET}=\frac{DE}{\tan XTD}=\frac{DE}{TX}.DX=\frac{AD.DX}{AH}=\frac{DE.DY }{2OD}=\frac{DB^2}{2OD} $, suy ra DT không đổi, suy ra ĐPCM

Em xem lại các biến đổi nhé.

Ta có thể làm như sau :
$$ DT = \frac{AE \cdot DX}{AX} = \frac{AD \cdot DX}{AH} = \cdots $$

kien10a1 16-04-2012 08:55 PM

Trích:

Nguyên văn bởi novae (Post 144931)
Em xem lại các biến đổi nhé.

Ta có thể làm như sau :
$$ DT = \frac{AE \cdot DX}{AX} = \frac{AD \cdot DX}{AH} = \cdots $$

Gọi Z là chân đường cao từ B thì ZXDC nội tiếp, suy ra $\widehat{DXC}=\widehat{DZC}=\widehat{DCA} \Rightarrow \Delta DXC\sim \Delta DCA\Rightarrow DX.DA=DC^2 $

12121993 16-04-2012 10:00 PM

Lời giải của mình cũng hơi tương tự. Sửa lại tên điểm cho giống ở trên để mọi người dễ theo dõi.
Kí hiệu $(XY) $ chỉ đường tròn đường kính $(XY). $
Gọi $M', N' $theo thứ tự là hình chiếu của $B, C $trên $AC, AB. H $ là trực tâm tam giác $ABC $. Ta có các bộ điểm thẳng hàng $(B, M, H, M') (C, N, H, N'). $Dễ thấy$ D, M, H, N $đồng viên.
$X $ là hình chiếu của $H $ trên $AD. $ Khi đó, $X $ thuộc $(HA), (HD). $
Xét 3 đường tròn $(BC), (HA), (HD) $ có các trục đẳng phương $M'N', XH, BC $đồng quy tại $I. $
Chú ý rẳng $AE, BM', CN' $đồng quy do đó $(BCEI)= -1 $. Qua phép chiếu xuyên tâm H ta có $H(MNEX)=-1 $ nên tứ giác $MENX $ điều hòa suy ra$ X, E, T $ thẳng hàng.
Gọi $S $ là giao điểm của $OD $ và $(HD) $ ta có $HS $ song song với $BC $và $D $ là trung điểm $BC $ suy ra $H(SDBC)=-1 $ hay $H(SDMN)=-1 $. Do đó $DMSN $ điều hòa suy ra $S, T, D $ thẳng hàng.
Dề thấy (bằng cộng góc)$ DM' $là tiếp tuyến của $(HA) $ dó đó $DC^2=DM'^2=DX.DA. $
Các tam giác$ AEX $và $ADH $ đồng dạng nên ta có $\frac{AE}{XA}=\frac{AD}{HA} $Ta có $TD $ song song với$ AH. $ Vì thế theo định lý Thales ta có $\frac{DT}{DX}=\frac{AE}{AX} $ suy ra $DT=DX.\frac{AE}{AX}=DX.\frac{DA}{AH}=\frac{DA.DX}{ AH}=\frac{DC^2}{2OD} $ không đổi.
Vậy$ T $ là điểm cố định.

conami 16-04-2012 11:10 PM

Bài này đề cho là mỗi máy bơm đặt trong 1 ô vuông và nó có thể tưới nước vào các ô có cạch chung với nó và cùng cot, cách nó đúng 1 ô, nhưng nếu như vậy thì có nghĩa là cái ô đặt cái máy bơm đó không được tưới !?
Mình nghĩ đáp số bài này ở cả 2 câu đều là n. Ta đặt máy bơm thành 1 hàng (không phải hàng trên cùng và dưới cùng)

kien10a1 16-04-2012 11:22 PM

Bài 2, Mình cũng nghĩ là cái ô đặt máy bơm không được chính nó tưới, đấy là cái khác của bài này với những bài kiểu tô maù đã có.
a, Ta xét $n\geq 2 $
Với m=4, xét bảng $4\times n $ đánh số thứ tự các cột từ 1 đến n. Ta gọi ô ở hàng i, cột j là (i;j)
Ta đánh dấu các ô (1;4k+1);(1;4k+2);(4;4k+3);(4;4k+4)
Hiển nhiên một máy bơm bất kỳ chỉ tưới được tối đa 1 ô trong các ô đánh dấu, suy ra có ít nhất n máy bơm.
Ta luôn chỉ ra được 1 cách đặt n máy bơm thỏa mãn: ví dụ đặt n máy bơm trên hàng thứ 2.

b, Ta chứng minh bằng quy nạp n=5k+2,3,4,5,6 thì có ít nhất 4k+2,3,4,5,6 máy
Gọi f(n) là số máy nhỏ nhất cần cho bảng $3\times n $
Ta có nhận xét: Để tưới nước cho bảng $3\times n $ mà cho phép dùng cả máy bơm ở ngoài bảng nhưng vẫn trong 3 hàng thì mất ít nhất f(n) máy bơm.
Thật vậy, với một máy bơm ở ngoài( gs sau cột thứ n) ta cho tương ứng một máy cùng hàng với nó và ở cột n-1, khi đó máy ở trong tưới hết các ô máy ở ngoài tưới, ta có thể bỏ máy ở ngoài.

Dễ kiểm tra với k=1.
Giả sử mệnh đề đúng với 1,2,...n.
Xét bảng $3\times (n+1) $
Xét 1 cách đặt máy thỏa mãn. Ta đi từ cột đầu tiên đi lại.
Xét cột đầu tiên của bảng, nếu nó có chứa ít nhất 2 máy bơm, ta xét n-1 cột cuối , các máy ở cột đầu không thể tưới đến đây, số máy để tưới hết ít nhất n-1 cột cuối trong bảng $3\times n $ theo nhận xét và giả thiết quy nạp nhỏ nhất là f(n-1), suy ra có ít nhất f(n-1)+2 máy. Tương tự như vậy ta thấy nếu trong a cột đầu mà đã có a+1 máy bơm thì phải dùng nhiều hơn f(n-1)+2 máy. Giả sử trong a cột đầu không bao giờ có đến a+1 máy bơm.
Cột đầu tiên có 0 máy thì suy ra ngay cột cạnh nó có 3 máy, 2 cột có 3 máy, loại.
Cột đầu tiên có đúng 1 máy, dễ thấy chỉ có duy nhất 1 vị trí có thể tưới nước cho nó là ô ngay cạnh, vậy ở đây phải có máy bơm, cột này có đúng 1 máy. Ta tiếp tục đi theo cột với thứ tự tăng dần.
Ta chứng minh nếu cứ trong a cột mà có không đến a máy thì sẽ còn cột phía sau. Ta cũng thấy việc lập luận chỉ phụ thuộc vào ta bắt đầu từ cột nào( tức là đi từ cột đầu có x máy cũng như đi từ cột nào đó có x máy)
Xét cột đầu tiên không có máy bơm nào, giả sử là cột thứ k. Theo giả sử cột trước nó có đúng 1 máy bơm, nghĩa là tưới được đúng 1 ô của nó, suy ra còn 2 ô chưa được tưới và phải có 2 máy bơm ở cột k+1( có 3 máy thì vi phạm)
Xét cột k+2, nếu nó không có máy bơm nào thì phải có cột k+3, xét 2 trường hợp:
+, cột k+3 có 1 máy thì chính vị trí của máy chưa được tưới và ta phải có 1 máy kề máy này ở cột k+4.
+, cột k+3 có 2 máy, ta lại tiếp tục xét như trên.
Suy ra nếu cột k không có máy thì phải có ít nhất 4 máy trong 4 cột tiếp theo
Do đó trong 5 cột liên tiếp ta luôn có ít nhất 4 máy bơm, suy ra $f(n+5) \geq f(n)+4 $

mathstarofvn 17-04-2012 11:50 AM

Theo mình bài toán tổ hợp phải hiểu là chúng ta chẳng cần tưới nước ô đặt máy bơm, đôi khi phải thực tế 1 tí là chẳng ai cần thiết đi bơm nước vào chỗ đặt máy bơm cả, như thế bài toán mới có vấn đề để nói và câu b đáp số ko còn là n nữa


Múi giờ GMT. Hiện tại là 02:24 AM.

Powered by: vBulletin Copyright ©2000-2024, Jelsoft Enterprises Ltd.

[page compression: 29.10 k/30.30 k (3.97%)]